Finite dimensional irreducible representations of a semisimple Lie Algebra separate points of the universal...











up vote
6
down vote

favorite
1












Let $mathfrak{g}$ be a semisimple Lie Algebra, and $U(mathfrak g)$ the universal enveloping algebra .



We know that for every representation $rho: mathfrak g to mathfrak{gl}(V)$, there exists a representation $tilde{rho} : U(mathfrak g) to mathfrak{gl}(V)$, such that $rho = tilde{rho} circ iota$, where $iota: mathfrak g to U(mathfrak g)$ is the natural inclusion. Besides that, using the standard notations, $tilde{rho}(X_1 cdot ldotscdot X_n) = rho(X_1) ldots rho(X_n).$



I'm very stuck in this problem




Question: Show that the finite dimensional irreducible representations of a semisimple Lie Algebra $mathfrak g$ separate points of the universal algebra $U(mathfrak g)$, i.e; if $a in U(mathfrak g)$ satisfies $tilde{rho}(a) =0$, for every irreducible representation $rho: mathfrak g to mathfrak{gl}(V)$, then $a=0$.




Can anyone help me?










share|cite|improve this question




















  • 1




    Where did this question arise?
    – Shaun
    Nov 17 at 4:21






  • 1




    I found this question on page 318 (chapter 11, problem 5) of the book "Algebras de Lie - Luiz A. B. San Martin"
    – Matheus Manzatto
    Nov 17 at 4:23

















up vote
6
down vote

favorite
1












Let $mathfrak{g}$ be a semisimple Lie Algebra, and $U(mathfrak g)$ the universal enveloping algebra .



We know that for every representation $rho: mathfrak g to mathfrak{gl}(V)$, there exists a representation $tilde{rho} : U(mathfrak g) to mathfrak{gl}(V)$, such that $rho = tilde{rho} circ iota$, where $iota: mathfrak g to U(mathfrak g)$ is the natural inclusion. Besides that, using the standard notations, $tilde{rho}(X_1 cdot ldotscdot X_n) = rho(X_1) ldots rho(X_n).$



I'm very stuck in this problem




Question: Show that the finite dimensional irreducible representations of a semisimple Lie Algebra $mathfrak g$ separate points of the universal algebra $U(mathfrak g)$, i.e; if $a in U(mathfrak g)$ satisfies $tilde{rho}(a) =0$, for every irreducible representation $rho: mathfrak g to mathfrak{gl}(V)$, then $a=0$.




Can anyone help me?










share|cite|improve this question




















  • 1




    Where did this question arise?
    – Shaun
    Nov 17 at 4:21






  • 1




    I found this question on page 318 (chapter 11, problem 5) of the book "Algebras de Lie - Luiz A. B. San Martin"
    – Matheus Manzatto
    Nov 17 at 4:23















up vote
6
down vote

favorite
1









up vote
6
down vote

favorite
1






1





Let $mathfrak{g}$ be a semisimple Lie Algebra, and $U(mathfrak g)$ the universal enveloping algebra .



We know that for every representation $rho: mathfrak g to mathfrak{gl}(V)$, there exists a representation $tilde{rho} : U(mathfrak g) to mathfrak{gl}(V)$, such that $rho = tilde{rho} circ iota$, where $iota: mathfrak g to U(mathfrak g)$ is the natural inclusion. Besides that, using the standard notations, $tilde{rho}(X_1 cdot ldotscdot X_n) = rho(X_1) ldots rho(X_n).$



I'm very stuck in this problem




Question: Show that the finite dimensional irreducible representations of a semisimple Lie Algebra $mathfrak g$ separate points of the universal algebra $U(mathfrak g)$, i.e; if $a in U(mathfrak g)$ satisfies $tilde{rho}(a) =0$, for every irreducible representation $rho: mathfrak g to mathfrak{gl}(V)$, then $a=0$.




Can anyone help me?










share|cite|improve this question















Let $mathfrak{g}$ be a semisimple Lie Algebra, and $U(mathfrak g)$ the universal enveloping algebra .



We know that for every representation $rho: mathfrak g to mathfrak{gl}(V)$, there exists a representation $tilde{rho} : U(mathfrak g) to mathfrak{gl}(V)$, such that $rho = tilde{rho} circ iota$, where $iota: mathfrak g to U(mathfrak g)$ is the natural inclusion. Besides that, using the standard notations, $tilde{rho}(X_1 cdot ldotscdot X_n) = rho(X_1) ldots rho(X_n).$



I'm very stuck in this problem




Question: Show that the finite dimensional irreducible representations of a semisimple Lie Algebra $mathfrak g$ separate points of the universal algebra $U(mathfrak g)$, i.e; if $a in U(mathfrak g)$ satisfies $tilde{rho}(a) =0$, for every irreducible representation $rho: mathfrak g to mathfrak{gl}(V)$, then $a=0$.




Can anyone help me?







abstract-algebra lie-algebras






share|cite|improve this question















share|cite|improve this question













share|cite|improve this question




share|cite|improve this question








edited Nov 17 at 22:34









Eran

1,178818




1,178818










asked Nov 17 at 2:29









Matheus Manzatto

1,3011523




1,3011523








  • 1




    Where did this question arise?
    – Shaun
    Nov 17 at 4:21






  • 1




    I found this question on page 318 (chapter 11, problem 5) of the book "Algebras de Lie - Luiz A. B. San Martin"
    – Matheus Manzatto
    Nov 17 at 4:23
















  • 1




    Where did this question arise?
    – Shaun
    Nov 17 at 4:21






  • 1




    I found this question on page 318 (chapter 11, problem 5) of the book "Algebras de Lie - Luiz A. B. San Martin"
    – Matheus Manzatto
    Nov 17 at 4:23










1




1




Where did this question arise?
– Shaun
Nov 17 at 4:21




Where did this question arise?
– Shaun
Nov 17 at 4:21




1




1




I found this question on page 318 (chapter 11, problem 5) of the book "Algebras de Lie - Luiz A. B. San Martin"
– Matheus Manzatto
Nov 17 at 4:23






I found this question on page 318 (chapter 11, problem 5) of the book "Algebras de Lie - Luiz A. B. San Martin"
– Matheus Manzatto
Nov 17 at 4:23












2 Answers
2






active

oldest

votes

















up vote
4
down vote



accepted










The following is an explicit argument building on the knowledge of the finite-dimensional irreducible representation of ${mathfrak g}$. At its heart is the non-degeneracy of the Shapovalov-form and the description of its determinant, but I tried to keep the exposition elementary.



Setup: Let ${mathfrak g}={mathfrak n}^-oplus{mathfrak h}oplus{mathfrak n}^+$ be a triangular decomposition of ${mathfrak g}$ with respect to a Cartan subalgebra ${mathfrak h}$ of ${mathfrak g}$ and a choice of positive roots $Phi^+subset{mathfrak h}^{ast}$. Further, let ${mathfrak b}:={mathfrak h}oplus{mathfrak n}^+$ be the associated Borel subalgebra. Finally, recall the PBW decomposition ${mathscr U}{mathfrak g}cong{mathscr U}{mathfrak n}^-otimes{mathscr U}{mathfrak h}otimes{mathscr U}{mathfrak n}^+$.



It is known (and not hard to show) that every finite-dimensional irreducible representation of ${mathfrak g}$ is uniquely of the form $L(lambda)=M(lambda)/N(lambda)$, where $lambdain{mathfrak h}^{ast}$ is dominant integral, i.e. $lambda(h_alpha)in{mathbb Z}_{geq 0}$ for all $alphainPhi^+$, and $M(lambda) := {mathscr U}{mathfrak g}otimes_{{mathscr U}{mathfrak b}} {mathbb C}_lambda$ for the $1$-dimensional ${mathscr U}{mathfrak b}$-module ${mathbb C}_lambda$ given by ${mathfrak n}^+{mathbb C}_lambda = {0}$ and ${mathfrak h}$ acting on ${mathbb C}_lambda$ via $lambda$.



It is important to get the idea of how $M(lambda)$ and $L(lambda)$ come about geometrically: The weight space diagram of $M(lambda)$ is a downwards directed cone rootes in $lambda$, while the one of $L(lambda)$ is its largest symmetric subset with respect to the Weyl group action. See here, for example, where the dotted lines indicate the weight cone of $M(lambda)$, and the solid area is where the weights of $L(lambda)$ live.



Let's consider the point separation for elements of ${mathscr U}{mathfrak n}^-$ first. For those, their action on $M(lambda)$ is very simple: As a ${mathscr U}{mathfrak n}^-$-module, $M(lambda)cong {mathscr U}{mathfrak n}^-$ with $1otimes 1mapsto 1$ because ${mathscr U}{mathfrak g}cong{mathscr U}{mathfrak n}^-otimes{mathscr U}{mathfrak h}otimes{mathscr U}{mathfrak n}^+$ by PBW. So no non-zero element of ${mathscr U}{mathfrak n}^-$ acts trivially on $M(lambda)$, because it doesn't kill the highest weight vector $1otimes 1$. The idea is now to make $lambdagg 0$ large enough, for any fixed element of ${mathscr U}{mathfrak n}^-setminus{0}$, so that this argument can be carried over to $L(lambda)$, showing that the element under consideration doesn't annihilate the highest weight vector. Intuitively, this should be possible, because the larger $lambda$ gets, the further 'away from' $lambda$ does the submodule $N(lambda)$ start that is annihilated from $M(lambda)$ when passing to $L(lambda)$.



Starting to be precise, you have the following:




Proposition: For any simple root $alphainDelta$, there is a unique embedding $M(s_alphacdotlambda)subset M(lambda)$, and $$L(lambda)=M(lambda)/sum_{alphainDelta} M(s_alphacdotlambda).$$




NB: Pursuing this further, you get the BGG resolution of $L(lambda)$ in terms of $M(wcdot lambda)$, with $win W$ in the $l(w)$-th syzygy.




Corollary: If $mupreceqlambda$ (i.e. $lambda-muin{mathbb Z}_+Phi^+$, so $mu$ is in the cone below $lambda$) but $lambda - mu = sum_{alphainDelta} c_alpha alpha$ with $c_alpha < lambda(h_alpha)$ for all $alphainDelta$, then the projection $M(lambda)_mutwoheadrightarrow L(lambda)_mu$ is an isomorphism.




In other words, it is only in the union of the 'shifted' cones rooted at $s_alphacdotlambda$ that $L(lambda)$ starts looking different from $M(lambda)$. This should be somewhat intuitive.



From that we get separation of points as follows:




Corollary: Let $theta = sum_{alphainDelta} c_alpha alpha$ with $c_alphain{mathbb Z}^+$, and suppose $yin{mathscr U}{mathfrak n}^-_{-theta}$; that is, $x$ is a sum of products $y_{alpha_{i_1}}cdots y_{alpha_{i_k}}$ such that $theta = sum_i alpha_{i_j}$. Then for any $lambdain{mathfrak h}^{ast}$ with $lambda(h_alpha)in{mathbb Z}^{> c_alpha}$ for all $alphainDelta$, $y.v_lambdaneq 0$ for the highest weight vector $v_lambda$ of $L(lambda)$. In particular, $xy$ doesn't act trivially on $L(lambda)$.




Proof: If $tilde{v}_lambda$ denotes the highest weight vector of $M(lambda)$, then by the previous proposition we have $y.tilde{v}_lambdain M(lambda)setminus N(lambda)$. In particular, $x$ acts nontrivially on the image $v_lambda$ of $tilde{v}_lambda$ in $L(lambda)$.




Corollary: Let $theta$, $yin{mathscr U}{mathfrak n}^-_{-theta}$ and $lambda$ be as before. Then there exists some $xin{mathscr U}{mathfrak n}^+_{theta}$ such that $(xy)_0(lambda)neq 0$, where $(xy)_0in {mathscr U}{mathfrak h}cong {mathscr P}({mathfrak h}^{ast})$ is the projection of $xyin{mathscr U}{mathfrak g}_{0}$ onto ${mathscr U}{mathfrak h}subset {mathscr U}{mathfrak g}_{0}$ with respect to the PBW decomposition.




Here, we used that ${mathscr U}{mathfrak h}cong {mathfrak S}({mathfrak h})cong {mathscr P}({mathfrak h}^{ast})$ can be viewed as the algebra of polynomial functions on ${mathfrak h}^{ast}$.



Proof: Since $y.v_lambdaneq 0$ in $L(lambda)$ and $L(lambda)$ is simple, we have $L(lambda)={mathscr U}{mathfrak g}.y.v_lambda={mathscr U}{mathfrak n}^-{mathscr U}{mathfrak b}.y.v_lambda$. In particular, there exists $xin {mathscr U}{mathfrak n}^+$ such that $(xy).v_lambdaneq 0$ in $L(lambda)_lambda$. For such $x$, we must have $(xy)_0neq 0$ since the $({mathscr U}{mathfrak g}){mathfrak n}^+$-component of $xy$ acts trivially on the highest weight vector $v_lambda$. Finally, note that $zin{mathscr U}{mathfrak h}$ acts on $v_lambda$ by $z(lambda)$.



In the previous corollary, the roles of $x$ and $y$ can be reversed:




Corollary: For $theta$, $lambda$ as before and $xin {mathscr U}{mathfrak n}^+_{theta}$, there exists an $yin {mathscr U}{mathfrak n}^-_{-theta}$ such that $(xy)_0(lambda)neq 0$.




Proof: Apply the corollary to $tau(y)in {mathscr U}{mathfrak n}^-_{-theta}$, where $tau:{mathscr U}{mathfrak g}^{text{opp}}to{mathscr U}{mathfrak g}$ is the auto-involution of ${mathscr U}{mathfrak g}$ swapping ${mathfrak n}^+$ and ${mathfrak n}^-$.




Theorem (Separation of Points): For any $zin {mathscr U}{mathfrak g}setminus{0}$ there exists a finite-dimensional $L(lambda)$ such that $z.L(lambda)neq 0$.




Proof: Assume $z=sum_theta y_theta h_theta x_theta$ where $x_thetain{mathscr U}{mathfrak n}^+_theta$ and $y_thetain{mathscr U}{mathfrak n}^-$, $h_thetain{mathscr U}{mathfrak h}$; in other words, you group PBW terms by the weight on the ${mathfrak n}$-side. Now, consider $theta$ maximal w.r.t. the ordering $lambdapreceqmu:Leftrightarrow mu-lambdain{mathbb Z}^+Phi^+$ such that $y_theta h_theta x_theta$ nonzero. Then, we know from our previous work that there's some $lambdagg 0$ such that for any $lambda^{prime}$ such $lambdapreceqlambda^{prime}$ there exists some $y^{prime}_thetain{mathscr U}{mathfrak n}^-_{-theta}$ (depending on $lambda^{prime}$) such that $(x_theta y^{prime}_theta)_0(lambda^{prime})neq 0$. Picking $lambda^{prime}$ large enough, we may assume that also $h_theta(lambda^{prime})neq 0$; this is because the polynomial $h_thetain {mathscr P}{mathfrak h}cong{mathscr P}({mathfrak h}^{ast})$ cannot vanish on the shifted half-lattice $lambda + {mathbb Z}^+Phi^+$. Putting everything together, in $L(lambda^{prime})$ we then have $(y_theta h_theta x_theta).(y^{prime}_theta v_{lambda^{prime}}) = h_theta(lambda^{prime}) (x_theta y^{prime}_theta)_0(lambda^{prime}) y_theta v_{lambda^{prime}}neq 0$, where for the last step we potentially have to enlarge $lambda^{prime}$ again. What about the other summands in $z$? They all annihilate $y^{prime}_theta v_{lambda^{prime}}$ because of the maximality of $theta$.






share|cite|improve this answer























  • Your answer was superb!
    – Matheus Manzatto
    Nov 23 at 15:35


















up vote
3
down vote














Theorem (Harish-Chandra 1949) for an arbitrary finite-dimensional Lie algebra over a field of characteristic, finite-dimensional representations separate points of the universal enveloping algebra.




This is a deep result, proved in Chap 2 of Dixmier's book "enveloping algebras". It has Ado's theorem as corollary.



In the semisimple case in characteristic zero, finite-dimensional representations split as direct sum of irreducible representations (Weyl), and hence the desired result follows: finite-dimensional irreducible representations separate points.



I don't know if Harish-Chandra's theorem is much easier in the semisimple case (for which Ado's theorem is trivial).



I'm not sure of the picture in finite characteristic, but this is probably tackled by work of Jacobson or so.






share|cite|improve this answer





















    Your Answer





    StackExchange.ifUsing("editor", function () {
    return StackExchange.using("mathjaxEditing", function () {
    StackExchange.MarkdownEditor.creationCallbacks.add(function (editor, postfix) {
    StackExchange.mathjaxEditing.prepareWmdForMathJax(editor, postfix, [["$", "$"], ["\\(","\\)"]]);
    });
    });
    }, "mathjax-editing");

    StackExchange.ready(function() {
    var channelOptions = {
    tags: "".split(" "),
    id: "69"
    };
    initTagRenderer("".split(" "), "".split(" "), channelOptions);

    StackExchange.using("externalEditor", function() {
    // Have to fire editor after snippets, if snippets enabled
    if (StackExchange.settings.snippets.snippetsEnabled) {
    StackExchange.using("snippets", function() {
    createEditor();
    });
    }
    else {
    createEditor();
    }
    });

    function createEditor() {
    StackExchange.prepareEditor({
    heartbeatType: 'answer',
    convertImagesToLinks: true,
    noModals: true,
    showLowRepImageUploadWarning: true,
    reputationToPostImages: 10,
    bindNavPrevention: true,
    postfix: "",
    imageUploader: {
    brandingHtml: "Powered by u003ca class="icon-imgur-white" href="https://imgur.com/"u003eu003c/au003e",
    contentPolicyHtml: "User contributions licensed under u003ca href="https://creativecommons.org/licenses/by-sa/3.0/"u003ecc by-sa 3.0 with attribution requiredu003c/au003e u003ca href="https://stackoverflow.com/legal/content-policy"u003e(content policy)u003c/au003e",
    allowUrls: true
    },
    noCode: true, onDemand: true,
    discardSelector: ".discard-answer"
    ,immediatelyShowMarkdownHelp:true
    });


    }
    });














    draft saved

    draft discarded


















    StackExchange.ready(
    function () {
    StackExchange.openid.initPostLogin('.new-post-login', 'https%3a%2f%2fmath.stackexchange.com%2fquestions%2f3001902%2ffinite-dimensional-irreducible-representations-of-a-semisimple-lie-algebra-separ%23new-answer', 'question_page');
    }
    );

    Post as a guest















    Required, but never shown

























    2 Answers
    2






    active

    oldest

    votes








    2 Answers
    2






    active

    oldest

    votes









    active

    oldest

    votes






    active

    oldest

    votes








    up vote
    4
    down vote



    accepted










    The following is an explicit argument building on the knowledge of the finite-dimensional irreducible representation of ${mathfrak g}$. At its heart is the non-degeneracy of the Shapovalov-form and the description of its determinant, but I tried to keep the exposition elementary.



    Setup: Let ${mathfrak g}={mathfrak n}^-oplus{mathfrak h}oplus{mathfrak n}^+$ be a triangular decomposition of ${mathfrak g}$ with respect to a Cartan subalgebra ${mathfrak h}$ of ${mathfrak g}$ and a choice of positive roots $Phi^+subset{mathfrak h}^{ast}$. Further, let ${mathfrak b}:={mathfrak h}oplus{mathfrak n}^+$ be the associated Borel subalgebra. Finally, recall the PBW decomposition ${mathscr U}{mathfrak g}cong{mathscr U}{mathfrak n}^-otimes{mathscr U}{mathfrak h}otimes{mathscr U}{mathfrak n}^+$.



    It is known (and not hard to show) that every finite-dimensional irreducible representation of ${mathfrak g}$ is uniquely of the form $L(lambda)=M(lambda)/N(lambda)$, where $lambdain{mathfrak h}^{ast}$ is dominant integral, i.e. $lambda(h_alpha)in{mathbb Z}_{geq 0}$ for all $alphainPhi^+$, and $M(lambda) := {mathscr U}{mathfrak g}otimes_{{mathscr U}{mathfrak b}} {mathbb C}_lambda$ for the $1$-dimensional ${mathscr U}{mathfrak b}$-module ${mathbb C}_lambda$ given by ${mathfrak n}^+{mathbb C}_lambda = {0}$ and ${mathfrak h}$ acting on ${mathbb C}_lambda$ via $lambda$.



    It is important to get the idea of how $M(lambda)$ and $L(lambda)$ come about geometrically: The weight space diagram of $M(lambda)$ is a downwards directed cone rootes in $lambda$, while the one of $L(lambda)$ is its largest symmetric subset with respect to the Weyl group action. See here, for example, where the dotted lines indicate the weight cone of $M(lambda)$, and the solid area is where the weights of $L(lambda)$ live.



    Let's consider the point separation for elements of ${mathscr U}{mathfrak n}^-$ first. For those, their action on $M(lambda)$ is very simple: As a ${mathscr U}{mathfrak n}^-$-module, $M(lambda)cong {mathscr U}{mathfrak n}^-$ with $1otimes 1mapsto 1$ because ${mathscr U}{mathfrak g}cong{mathscr U}{mathfrak n}^-otimes{mathscr U}{mathfrak h}otimes{mathscr U}{mathfrak n}^+$ by PBW. So no non-zero element of ${mathscr U}{mathfrak n}^-$ acts trivially on $M(lambda)$, because it doesn't kill the highest weight vector $1otimes 1$. The idea is now to make $lambdagg 0$ large enough, for any fixed element of ${mathscr U}{mathfrak n}^-setminus{0}$, so that this argument can be carried over to $L(lambda)$, showing that the element under consideration doesn't annihilate the highest weight vector. Intuitively, this should be possible, because the larger $lambda$ gets, the further 'away from' $lambda$ does the submodule $N(lambda)$ start that is annihilated from $M(lambda)$ when passing to $L(lambda)$.



    Starting to be precise, you have the following:




    Proposition: For any simple root $alphainDelta$, there is a unique embedding $M(s_alphacdotlambda)subset M(lambda)$, and $$L(lambda)=M(lambda)/sum_{alphainDelta} M(s_alphacdotlambda).$$




    NB: Pursuing this further, you get the BGG resolution of $L(lambda)$ in terms of $M(wcdot lambda)$, with $win W$ in the $l(w)$-th syzygy.




    Corollary: If $mupreceqlambda$ (i.e. $lambda-muin{mathbb Z}_+Phi^+$, so $mu$ is in the cone below $lambda$) but $lambda - mu = sum_{alphainDelta} c_alpha alpha$ with $c_alpha < lambda(h_alpha)$ for all $alphainDelta$, then the projection $M(lambda)_mutwoheadrightarrow L(lambda)_mu$ is an isomorphism.




    In other words, it is only in the union of the 'shifted' cones rooted at $s_alphacdotlambda$ that $L(lambda)$ starts looking different from $M(lambda)$. This should be somewhat intuitive.



    From that we get separation of points as follows:




    Corollary: Let $theta = sum_{alphainDelta} c_alpha alpha$ with $c_alphain{mathbb Z}^+$, and suppose $yin{mathscr U}{mathfrak n}^-_{-theta}$; that is, $x$ is a sum of products $y_{alpha_{i_1}}cdots y_{alpha_{i_k}}$ such that $theta = sum_i alpha_{i_j}$. Then for any $lambdain{mathfrak h}^{ast}$ with $lambda(h_alpha)in{mathbb Z}^{> c_alpha}$ for all $alphainDelta$, $y.v_lambdaneq 0$ for the highest weight vector $v_lambda$ of $L(lambda)$. In particular, $xy$ doesn't act trivially on $L(lambda)$.




    Proof: If $tilde{v}_lambda$ denotes the highest weight vector of $M(lambda)$, then by the previous proposition we have $y.tilde{v}_lambdain M(lambda)setminus N(lambda)$. In particular, $x$ acts nontrivially on the image $v_lambda$ of $tilde{v}_lambda$ in $L(lambda)$.




    Corollary: Let $theta$, $yin{mathscr U}{mathfrak n}^-_{-theta}$ and $lambda$ be as before. Then there exists some $xin{mathscr U}{mathfrak n}^+_{theta}$ such that $(xy)_0(lambda)neq 0$, where $(xy)_0in {mathscr U}{mathfrak h}cong {mathscr P}({mathfrak h}^{ast})$ is the projection of $xyin{mathscr U}{mathfrak g}_{0}$ onto ${mathscr U}{mathfrak h}subset {mathscr U}{mathfrak g}_{0}$ with respect to the PBW decomposition.




    Here, we used that ${mathscr U}{mathfrak h}cong {mathfrak S}({mathfrak h})cong {mathscr P}({mathfrak h}^{ast})$ can be viewed as the algebra of polynomial functions on ${mathfrak h}^{ast}$.



    Proof: Since $y.v_lambdaneq 0$ in $L(lambda)$ and $L(lambda)$ is simple, we have $L(lambda)={mathscr U}{mathfrak g}.y.v_lambda={mathscr U}{mathfrak n}^-{mathscr U}{mathfrak b}.y.v_lambda$. In particular, there exists $xin {mathscr U}{mathfrak n}^+$ such that $(xy).v_lambdaneq 0$ in $L(lambda)_lambda$. For such $x$, we must have $(xy)_0neq 0$ since the $({mathscr U}{mathfrak g}){mathfrak n}^+$-component of $xy$ acts trivially on the highest weight vector $v_lambda$. Finally, note that $zin{mathscr U}{mathfrak h}$ acts on $v_lambda$ by $z(lambda)$.



    In the previous corollary, the roles of $x$ and $y$ can be reversed:




    Corollary: For $theta$, $lambda$ as before and $xin {mathscr U}{mathfrak n}^+_{theta}$, there exists an $yin {mathscr U}{mathfrak n}^-_{-theta}$ such that $(xy)_0(lambda)neq 0$.




    Proof: Apply the corollary to $tau(y)in {mathscr U}{mathfrak n}^-_{-theta}$, where $tau:{mathscr U}{mathfrak g}^{text{opp}}to{mathscr U}{mathfrak g}$ is the auto-involution of ${mathscr U}{mathfrak g}$ swapping ${mathfrak n}^+$ and ${mathfrak n}^-$.




    Theorem (Separation of Points): For any $zin {mathscr U}{mathfrak g}setminus{0}$ there exists a finite-dimensional $L(lambda)$ such that $z.L(lambda)neq 0$.




    Proof: Assume $z=sum_theta y_theta h_theta x_theta$ where $x_thetain{mathscr U}{mathfrak n}^+_theta$ and $y_thetain{mathscr U}{mathfrak n}^-$, $h_thetain{mathscr U}{mathfrak h}$; in other words, you group PBW terms by the weight on the ${mathfrak n}$-side. Now, consider $theta$ maximal w.r.t. the ordering $lambdapreceqmu:Leftrightarrow mu-lambdain{mathbb Z}^+Phi^+$ such that $y_theta h_theta x_theta$ nonzero. Then, we know from our previous work that there's some $lambdagg 0$ such that for any $lambda^{prime}$ such $lambdapreceqlambda^{prime}$ there exists some $y^{prime}_thetain{mathscr U}{mathfrak n}^-_{-theta}$ (depending on $lambda^{prime}$) such that $(x_theta y^{prime}_theta)_0(lambda^{prime})neq 0$. Picking $lambda^{prime}$ large enough, we may assume that also $h_theta(lambda^{prime})neq 0$; this is because the polynomial $h_thetain {mathscr P}{mathfrak h}cong{mathscr P}({mathfrak h}^{ast})$ cannot vanish on the shifted half-lattice $lambda + {mathbb Z}^+Phi^+$. Putting everything together, in $L(lambda^{prime})$ we then have $(y_theta h_theta x_theta).(y^{prime}_theta v_{lambda^{prime}}) = h_theta(lambda^{prime}) (x_theta y^{prime}_theta)_0(lambda^{prime}) y_theta v_{lambda^{prime}}neq 0$, where for the last step we potentially have to enlarge $lambda^{prime}$ again. What about the other summands in $z$? They all annihilate $y^{prime}_theta v_{lambda^{prime}}$ because of the maximality of $theta$.






    share|cite|improve this answer























    • Your answer was superb!
      – Matheus Manzatto
      Nov 23 at 15:35















    up vote
    4
    down vote



    accepted










    The following is an explicit argument building on the knowledge of the finite-dimensional irreducible representation of ${mathfrak g}$. At its heart is the non-degeneracy of the Shapovalov-form and the description of its determinant, but I tried to keep the exposition elementary.



    Setup: Let ${mathfrak g}={mathfrak n}^-oplus{mathfrak h}oplus{mathfrak n}^+$ be a triangular decomposition of ${mathfrak g}$ with respect to a Cartan subalgebra ${mathfrak h}$ of ${mathfrak g}$ and a choice of positive roots $Phi^+subset{mathfrak h}^{ast}$. Further, let ${mathfrak b}:={mathfrak h}oplus{mathfrak n}^+$ be the associated Borel subalgebra. Finally, recall the PBW decomposition ${mathscr U}{mathfrak g}cong{mathscr U}{mathfrak n}^-otimes{mathscr U}{mathfrak h}otimes{mathscr U}{mathfrak n}^+$.



    It is known (and not hard to show) that every finite-dimensional irreducible representation of ${mathfrak g}$ is uniquely of the form $L(lambda)=M(lambda)/N(lambda)$, where $lambdain{mathfrak h}^{ast}$ is dominant integral, i.e. $lambda(h_alpha)in{mathbb Z}_{geq 0}$ for all $alphainPhi^+$, and $M(lambda) := {mathscr U}{mathfrak g}otimes_{{mathscr U}{mathfrak b}} {mathbb C}_lambda$ for the $1$-dimensional ${mathscr U}{mathfrak b}$-module ${mathbb C}_lambda$ given by ${mathfrak n}^+{mathbb C}_lambda = {0}$ and ${mathfrak h}$ acting on ${mathbb C}_lambda$ via $lambda$.



    It is important to get the idea of how $M(lambda)$ and $L(lambda)$ come about geometrically: The weight space diagram of $M(lambda)$ is a downwards directed cone rootes in $lambda$, while the one of $L(lambda)$ is its largest symmetric subset with respect to the Weyl group action. See here, for example, where the dotted lines indicate the weight cone of $M(lambda)$, and the solid area is where the weights of $L(lambda)$ live.



    Let's consider the point separation for elements of ${mathscr U}{mathfrak n}^-$ first. For those, their action on $M(lambda)$ is very simple: As a ${mathscr U}{mathfrak n}^-$-module, $M(lambda)cong {mathscr U}{mathfrak n}^-$ with $1otimes 1mapsto 1$ because ${mathscr U}{mathfrak g}cong{mathscr U}{mathfrak n}^-otimes{mathscr U}{mathfrak h}otimes{mathscr U}{mathfrak n}^+$ by PBW. So no non-zero element of ${mathscr U}{mathfrak n}^-$ acts trivially on $M(lambda)$, because it doesn't kill the highest weight vector $1otimes 1$. The idea is now to make $lambdagg 0$ large enough, for any fixed element of ${mathscr U}{mathfrak n}^-setminus{0}$, so that this argument can be carried over to $L(lambda)$, showing that the element under consideration doesn't annihilate the highest weight vector. Intuitively, this should be possible, because the larger $lambda$ gets, the further 'away from' $lambda$ does the submodule $N(lambda)$ start that is annihilated from $M(lambda)$ when passing to $L(lambda)$.



    Starting to be precise, you have the following:




    Proposition: For any simple root $alphainDelta$, there is a unique embedding $M(s_alphacdotlambda)subset M(lambda)$, and $$L(lambda)=M(lambda)/sum_{alphainDelta} M(s_alphacdotlambda).$$




    NB: Pursuing this further, you get the BGG resolution of $L(lambda)$ in terms of $M(wcdot lambda)$, with $win W$ in the $l(w)$-th syzygy.




    Corollary: If $mupreceqlambda$ (i.e. $lambda-muin{mathbb Z}_+Phi^+$, so $mu$ is in the cone below $lambda$) but $lambda - mu = sum_{alphainDelta} c_alpha alpha$ with $c_alpha < lambda(h_alpha)$ for all $alphainDelta$, then the projection $M(lambda)_mutwoheadrightarrow L(lambda)_mu$ is an isomorphism.




    In other words, it is only in the union of the 'shifted' cones rooted at $s_alphacdotlambda$ that $L(lambda)$ starts looking different from $M(lambda)$. This should be somewhat intuitive.



    From that we get separation of points as follows:




    Corollary: Let $theta = sum_{alphainDelta} c_alpha alpha$ with $c_alphain{mathbb Z}^+$, and suppose $yin{mathscr U}{mathfrak n}^-_{-theta}$; that is, $x$ is a sum of products $y_{alpha_{i_1}}cdots y_{alpha_{i_k}}$ such that $theta = sum_i alpha_{i_j}$. Then for any $lambdain{mathfrak h}^{ast}$ with $lambda(h_alpha)in{mathbb Z}^{> c_alpha}$ for all $alphainDelta$, $y.v_lambdaneq 0$ for the highest weight vector $v_lambda$ of $L(lambda)$. In particular, $xy$ doesn't act trivially on $L(lambda)$.




    Proof: If $tilde{v}_lambda$ denotes the highest weight vector of $M(lambda)$, then by the previous proposition we have $y.tilde{v}_lambdain M(lambda)setminus N(lambda)$. In particular, $x$ acts nontrivially on the image $v_lambda$ of $tilde{v}_lambda$ in $L(lambda)$.




    Corollary: Let $theta$, $yin{mathscr U}{mathfrak n}^-_{-theta}$ and $lambda$ be as before. Then there exists some $xin{mathscr U}{mathfrak n}^+_{theta}$ such that $(xy)_0(lambda)neq 0$, where $(xy)_0in {mathscr U}{mathfrak h}cong {mathscr P}({mathfrak h}^{ast})$ is the projection of $xyin{mathscr U}{mathfrak g}_{0}$ onto ${mathscr U}{mathfrak h}subset {mathscr U}{mathfrak g}_{0}$ with respect to the PBW decomposition.




    Here, we used that ${mathscr U}{mathfrak h}cong {mathfrak S}({mathfrak h})cong {mathscr P}({mathfrak h}^{ast})$ can be viewed as the algebra of polynomial functions on ${mathfrak h}^{ast}$.



    Proof: Since $y.v_lambdaneq 0$ in $L(lambda)$ and $L(lambda)$ is simple, we have $L(lambda)={mathscr U}{mathfrak g}.y.v_lambda={mathscr U}{mathfrak n}^-{mathscr U}{mathfrak b}.y.v_lambda$. In particular, there exists $xin {mathscr U}{mathfrak n}^+$ such that $(xy).v_lambdaneq 0$ in $L(lambda)_lambda$. For such $x$, we must have $(xy)_0neq 0$ since the $({mathscr U}{mathfrak g}){mathfrak n}^+$-component of $xy$ acts trivially on the highest weight vector $v_lambda$. Finally, note that $zin{mathscr U}{mathfrak h}$ acts on $v_lambda$ by $z(lambda)$.



    In the previous corollary, the roles of $x$ and $y$ can be reversed:




    Corollary: For $theta$, $lambda$ as before and $xin {mathscr U}{mathfrak n}^+_{theta}$, there exists an $yin {mathscr U}{mathfrak n}^-_{-theta}$ such that $(xy)_0(lambda)neq 0$.




    Proof: Apply the corollary to $tau(y)in {mathscr U}{mathfrak n}^-_{-theta}$, where $tau:{mathscr U}{mathfrak g}^{text{opp}}to{mathscr U}{mathfrak g}$ is the auto-involution of ${mathscr U}{mathfrak g}$ swapping ${mathfrak n}^+$ and ${mathfrak n}^-$.




    Theorem (Separation of Points): For any $zin {mathscr U}{mathfrak g}setminus{0}$ there exists a finite-dimensional $L(lambda)$ such that $z.L(lambda)neq 0$.




    Proof: Assume $z=sum_theta y_theta h_theta x_theta$ where $x_thetain{mathscr U}{mathfrak n}^+_theta$ and $y_thetain{mathscr U}{mathfrak n}^-$, $h_thetain{mathscr U}{mathfrak h}$; in other words, you group PBW terms by the weight on the ${mathfrak n}$-side. Now, consider $theta$ maximal w.r.t. the ordering $lambdapreceqmu:Leftrightarrow mu-lambdain{mathbb Z}^+Phi^+$ such that $y_theta h_theta x_theta$ nonzero. Then, we know from our previous work that there's some $lambdagg 0$ such that for any $lambda^{prime}$ such $lambdapreceqlambda^{prime}$ there exists some $y^{prime}_thetain{mathscr U}{mathfrak n}^-_{-theta}$ (depending on $lambda^{prime}$) such that $(x_theta y^{prime}_theta)_0(lambda^{prime})neq 0$. Picking $lambda^{prime}$ large enough, we may assume that also $h_theta(lambda^{prime})neq 0$; this is because the polynomial $h_thetain {mathscr P}{mathfrak h}cong{mathscr P}({mathfrak h}^{ast})$ cannot vanish on the shifted half-lattice $lambda + {mathbb Z}^+Phi^+$. Putting everything together, in $L(lambda^{prime})$ we then have $(y_theta h_theta x_theta).(y^{prime}_theta v_{lambda^{prime}}) = h_theta(lambda^{prime}) (x_theta y^{prime}_theta)_0(lambda^{prime}) y_theta v_{lambda^{prime}}neq 0$, where for the last step we potentially have to enlarge $lambda^{prime}$ again. What about the other summands in $z$? They all annihilate $y^{prime}_theta v_{lambda^{prime}}$ because of the maximality of $theta$.






    share|cite|improve this answer























    • Your answer was superb!
      – Matheus Manzatto
      Nov 23 at 15:35













    up vote
    4
    down vote



    accepted







    up vote
    4
    down vote



    accepted






    The following is an explicit argument building on the knowledge of the finite-dimensional irreducible representation of ${mathfrak g}$. At its heart is the non-degeneracy of the Shapovalov-form and the description of its determinant, but I tried to keep the exposition elementary.



    Setup: Let ${mathfrak g}={mathfrak n}^-oplus{mathfrak h}oplus{mathfrak n}^+$ be a triangular decomposition of ${mathfrak g}$ with respect to a Cartan subalgebra ${mathfrak h}$ of ${mathfrak g}$ and a choice of positive roots $Phi^+subset{mathfrak h}^{ast}$. Further, let ${mathfrak b}:={mathfrak h}oplus{mathfrak n}^+$ be the associated Borel subalgebra. Finally, recall the PBW decomposition ${mathscr U}{mathfrak g}cong{mathscr U}{mathfrak n}^-otimes{mathscr U}{mathfrak h}otimes{mathscr U}{mathfrak n}^+$.



    It is known (and not hard to show) that every finite-dimensional irreducible representation of ${mathfrak g}$ is uniquely of the form $L(lambda)=M(lambda)/N(lambda)$, where $lambdain{mathfrak h}^{ast}$ is dominant integral, i.e. $lambda(h_alpha)in{mathbb Z}_{geq 0}$ for all $alphainPhi^+$, and $M(lambda) := {mathscr U}{mathfrak g}otimes_{{mathscr U}{mathfrak b}} {mathbb C}_lambda$ for the $1$-dimensional ${mathscr U}{mathfrak b}$-module ${mathbb C}_lambda$ given by ${mathfrak n}^+{mathbb C}_lambda = {0}$ and ${mathfrak h}$ acting on ${mathbb C}_lambda$ via $lambda$.



    It is important to get the idea of how $M(lambda)$ and $L(lambda)$ come about geometrically: The weight space diagram of $M(lambda)$ is a downwards directed cone rootes in $lambda$, while the one of $L(lambda)$ is its largest symmetric subset with respect to the Weyl group action. See here, for example, where the dotted lines indicate the weight cone of $M(lambda)$, and the solid area is where the weights of $L(lambda)$ live.



    Let's consider the point separation for elements of ${mathscr U}{mathfrak n}^-$ first. For those, their action on $M(lambda)$ is very simple: As a ${mathscr U}{mathfrak n}^-$-module, $M(lambda)cong {mathscr U}{mathfrak n}^-$ with $1otimes 1mapsto 1$ because ${mathscr U}{mathfrak g}cong{mathscr U}{mathfrak n}^-otimes{mathscr U}{mathfrak h}otimes{mathscr U}{mathfrak n}^+$ by PBW. So no non-zero element of ${mathscr U}{mathfrak n}^-$ acts trivially on $M(lambda)$, because it doesn't kill the highest weight vector $1otimes 1$. The idea is now to make $lambdagg 0$ large enough, for any fixed element of ${mathscr U}{mathfrak n}^-setminus{0}$, so that this argument can be carried over to $L(lambda)$, showing that the element under consideration doesn't annihilate the highest weight vector. Intuitively, this should be possible, because the larger $lambda$ gets, the further 'away from' $lambda$ does the submodule $N(lambda)$ start that is annihilated from $M(lambda)$ when passing to $L(lambda)$.



    Starting to be precise, you have the following:




    Proposition: For any simple root $alphainDelta$, there is a unique embedding $M(s_alphacdotlambda)subset M(lambda)$, and $$L(lambda)=M(lambda)/sum_{alphainDelta} M(s_alphacdotlambda).$$




    NB: Pursuing this further, you get the BGG resolution of $L(lambda)$ in terms of $M(wcdot lambda)$, with $win W$ in the $l(w)$-th syzygy.




    Corollary: If $mupreceqlambda$ (i.e. $lambda-muin{mathbb Z}_+Phi^+$, so $mu$ is in the cone below $lambda$) but $lambda - mu = sum_{alphainDelta} c_alpha alpha$ with $c_alpha < lambda(h_alpha)$ for all $alphainDelta$, then the projection $M(lambda)_mutwoheadrightarrow L(lambda)_mu$ is an isomorphism.




    In other words, it is only in the union of the 'shifted' cones rooted at $s_alphacdotlambda$ that $L(lambda)$ starts looking different from $M(lambda)$. This should be somewhat intuitive.



    From that we get separation of points as follows:




    Corollary: Let $theta = sum_{alphainDelta} c_alpha alpha$ with $c_alphain{mathbb Z}^+$, and suppose $yin{mathscr U}{mathfrak n}^-_{-theta}$; that is, $x$ is a sum of products $y_{alpha_{i_1}}cdots y_{alpha_{i_k}}$ such that $theta = sum_i alpha_{i_j}$. Then for any $lambdain{mathfrak h}^{ast}$ with $lambda(h_alpha)in{mathbb Z}^{> c_alpha}$ for all $alphainDelta$, $y.v_lambdaneq 0$ for the highest weight vector $v_lambda$ of $L(lambda)$. In particular, $xy$ doesn't act trivially on $L(lambda)$.




    Proof: If $tilde{v}_lambda$ denotes the highest weight vector of $M(lambda)$, then by the previous proposition we have $y.tilde{v}_lambdain M(lambda)setminus N(lambda)$. In particular, $x$ acts nontrivially on the image $v_lambda$ of $tilde{v}_lambda$ in $L(lambda)$.




    Corollary: Let $theta$, $yin{mathscr U}{mathfrak n}^-_{-theta}$ and $lambda$ be as before. Then there exists some $xin{mathscr U}{mathfrak n}^+_{theta}$ such that $(xy)_0(lambda)neq 0$, where $(xy)_0in {mathscr U}{mathfrak h}cong {mathscr P}({mathfrak h}^{ast})$ is the projection of $xyin{mathscr U}{mathfrak g}_{0}$ onto ${mathscr U}{mathfrak h}subset {mathscr U}{mathfrak g}_{0}$ with respect to the PBW decomposition.




    Here, we used that ${mathscr U}{mathfrak h}cong {mathfrak S}({mathfrak h})cong {mathscr P}({mathfrak h}^{ast})$ can be viewed as the algebra of polynomial functions on ${mathfrak h}^{ast}$.



    Proof: Since $y.v_lambdaneq 0$ in $L(lambda)$ and $L(lambda)$ is simple, we have $L(lambda)={mathscr U}{mathfrak g}.y.v_lambda={mathscr U}{mathfrak n}^-{mathscr U}{mathfrak b}.y.v_lambda$. In particular, there exists $xin {mathscr U}{mathfrak n}^+$ such that $(xy).v_lambdaneq 0$ in $L(lambda)_lambda$. For such $x$, we must have $(xy)_0neq 0$ since the $({mathscr U}{mathfrak g}){mathfrak n}^+$-component of $xy$ acts trivially on the highest weight vector $v_lambda$. Finally, note that $zin{mathscr U}{mathfrak h}$ acts on $v_lambda$ by $z(lambda)$.



    In the previous corollary, the roles of $x$ and $y$ can be reversed:




    Corollary: For $theta$, $lambda$ as before and $xin {mathscr U}{mathfrak n}^+_{theta}$, there exists an $yin {mathscr U}{mathfrak n}^-_{-theta}$ such that $(xy)_0(lambda)neq 0$.




    Proof: Apply the corollary to $tau(y)in {mathscr U}{mathfrak n}^-_{-theta}$, where $tau:{mathscr U}{mathfrak g}^{text{opp}}to{mathscr U}{mathfrak g}$ is the auto-involution of ${mathscr U}{mathfrak g}$ swapping ${mathfrak n}^+$ and ${mathfrak n}^-$.




    Theorem (Separation of Points): For any $zin {mathscr U}{mathfrak g}setminus{0}$ there exists a finite-dimensional $L(lambda)$ such that $z.L(lambda)neq 0$.




    Proof: Assume $z=sum_theta y_theta h_theta x_theta$ where $x_thetain{mathscr U}{mathfrak n}^+_theta$ and $y_thetain{mathscr U}{mathfrak n}^-$, $h_thetain{mathscr U}{mathfrak h}$; in other words, you group PBW terms by the weight on the ${mathfrak n}$-side. Now, consider $theta$ maximal w.r.t. the ordering $lambdapreceqmu:Leftrightarrow mu-lambdain{mathbb Z}^+Phi^+$ such that $y_theta h_theta x_theta$ nonzero. Then, we know from our previous work that there's some $lambdagg 0$ such that for any $lambda^{prime}$ such $lambdapreceqlambda^{prime}$ there exists some $y^{prime}_thetain{mathscr U}{mathfrak n}^-_{-theta}$ (depending on $lambda^{prime}$) such that $(x_theta y^{prime}_theta)_0(lambda^{prime})neq 0$. Picking $lambda^{prime}$ large enough, we may assume that also $h_theta(lambda^{prime})neq 0$; this is because the polynomial $h_thetain {mathscr P}{mathfrak h}cong{mathscr P}({mathfrak h}^{ast})$ cannot vanish on the shifted half-lattice $lambda + {mathbb Z}^+Phi^+$. Putting everything together, in $L(lambda^{prime})$ we then have $(y_theta h_theta x_theta).(y^{prime}_theta v_{lambda^{prime}}) = h_theta(lambda^{prime}) (x_theta y^{prime}_theta)_0(lambda^{prime}) y_theta v_{lambda^{prime}}neq 0$, where for the last step we potentially have to enlarge $lambda^{prime}$ again. What about the other summands in $z$? They all annihilate $y^{prime}_theta v_{lambda^{prime}}$ because of the maximality of $theta$.






    share|cite|improve this answer














    The following is an explicit argument building on the knowledge of the finite-dimensional irreducible representation of ${mathfrak g}$. At its heart is the non-degeneracy of the Shapovalov-form and the description of its determinant, but I tried to keep the exposition elementary.



    Setup: Let ${mathfrak g}={mathfrak n}^-oplus{mathfrak h}oplus{mathfrak n}^+$ be a triangular decomposition of ${mathfrak g}$ with respect to a Cartan subalgebra ${mathfrak h}$ of ${mathfrak g}$ and a choice of positive roots $Phi^+subset{mathfrak h}^{ast}$. Further, let ${mathfrak b}:={mathfrak h}oplus{mathfrak n}^+$ be the associated Borel subalgebra. Finally, recall the PBW decomposition ${mathscr U}{mathfrak g}cong{mathscr U}{mathfrak n}^-otimes{mathscr U}{mathfrak h}otimes{mathscr U}{mathfrak n}^+$.



    It is known (and not hard to show) that every finite-dimensional irreducible representation of ${mathfrak g}$ is uniquely of the form $L(lambda)=M(lambda)/N(lambda)$, where $lambdain{mathfrak h}^{ast}$ is dominant integral, i.e. $lambda(h_alpha)in{mathbb Z}_{geq 0}$ for all $alphainPhi^+$, and $M(lambda) := {mathscr U}{mathfrak g}otimes_{{mathscr U}{mathfrak b}} {mathbb C}_lambda$ for the $1$-dimensional ${mathscr U}{mathfrak b}$-module ${mathbb C}_lambda$ given by ${mathfrak n}^+{mathbb C}_lambda = {0}$ and ${mathfrak h}$ acting on ${mathbb C}_lambda$ via $lambda$.



    It is important to get the idea of how $M(lambda)$ and $L(lambda)$ come about geometrically: The weight space diagram of $M(lambda)$ is a downwards directed cone rootes in $lambda$, while the one of $L(lambda)$ is its largest symmetric subset with respect to the Weyl group action. See here, for example, where the dotted lines indicate the weight cone of $M(lambda)$, and the solid area is where the weights of $L(lambda)$ live.



    Let's consider the point separation for elements of ${mathscr U}{mathfrak n}^-$ first. For those, their action on $M(lambda)$ is very simple: As a ${mathscr U}{mathfrak n}^-$-module, $M(lambda)cong {mathscr U}{mathfrak n}^-$ with $1otimes 1mapsto 1$ because ${mathscr U}{mathfrak g}cong{mathscr U}{mathfrak n}^-otimes{mathscr U}{mathfrak h}otimes{mathscr U}{mathfrak n}^+$ by PBW. So no non-zero element of ${mathscr U}{mathfrak n}^-$ acts trivially on $M(lambda)$, because it doesn't kill the highest weight vector $1otimes 1$. The idea is now to make $lambdagg 0$ large enough, for any fixed element of ${mathscr U}{mathfrak n}^-setminus{0}$, so that this argument can be carried over to $L(lambda)$, showing that the element under consideration doesn't annihilate the highest weight vector. Intuitively, this should be possible, because the larger $lambda$ gets, the further 'away from' $lambda$ does the submodule $N(lambda)$ start that is annihilated from $M(lambda)$ when passing to $L(lambda)$.



    Starting to be precise, you have the following:




    Proposition: For any simple root $alphainDelta$, there is a unique embedding $M(s_alphacdotlambda)subset M(lambda)$, and $$L(lambda)=M(lambda)/sum_{alphainDelta} M(s_alphacdotlambda).$$




    NB: Pursuing this further, you get the BGG resolution of $L(lambda)$ in terms of $M(wcdot lambda)$, with $win W$ in the $l(w)$-th syzygy.




    Corollary: If $mupreceqlambda$ (i.e. $lambda-muin{mathbb Z}_+Phi^+$, so $mu$ is in the cone below $lambda$) but $lambda - mu = sum_{alphainDelta} c_alpha alpha$ with $c_alpha < lambda(h_alpha)$ for all $alphainDelta$, then the projection $M(lambda)_mutwoheadrightarrow L(lambda)_mu$ is an isomorphism.




    In other words, it is only in the union of the 'shifted' cones rooted at $s_alphacdotlambda$ that $L(lambda)$ starts looking different from $M(lambda)$. This should be somewhat intuitive.



    From that we get separation of points as follows:




    Corollary: Let $theta = sum_{alphainDelta} c_alpha alpha$ with $c_alphain{mathbb Z}^+$, and suppose $yin{mathscr U}{mathfrak n}^-_{-theta}$; that is, $x$ is a sum of products $y_{alpha_{i_1}}cdots y_{alpha_{i_k}}$ such that $theta = sum_i alpha_{i_j}$. Then for any $lambdain{mathfrak h}^{ast}$ with $lambda(h_alpha)in{mathbb Z}^{> c_alpha}$ for all $alphainDelta$, $y.v_lambdaneq 0$ for the highest weight vector $v_lambda$ of $L(lambda)$. In particular, $xy$ doesn't act trivially on $L(lambda)$.




    Proof: If $tilde{v}_lambda$ denotes the highest weight vector of $M(lambda)$, then by the previous proposition we have $y.tilde{v}_lambdain M(lambda)setminus N(lambda)$. In particular, $x$ acts nontrivially on the image $v_lambda$ of $tilde{v}_lambda$ in $L(lambda)$.




    Corollary: Let $theta$, $yin{mathscr U}{mathfrak n}^-_{-theta}$ and $lambda$ be as before. Then there exists some $xin{mathscr U}{mathfrak n}^+_{theta}$ such that $(xy)_0(lambda)neq 0$, where $(xy)_0in {mathscr U}{mathfrak h}cong {mathscr P}({mathfrak h}^{ast})$ is the projection of $xyin{mathscr U}{mathfrak g}_{0}$ onto ${mathscr U}{mathfrak h}subset {mathscr U}{mathfrak g}_{0}$ with respect to the PBW decomposition.




    Here, we used that ${mathscr U}{mathfrak h}cong {mathfrak S}({mathfrak h})cong {mathscr P}({mathfrak h}^{ast})$ can be viewed as the algebra of polynomial functions on ${mathfrak h}^{ast}$.



    Proof: Since $y.v_lambdaneq 0$ in $L(lambda)$ and $L(lambda)$ is simple, we have $L(lambda)={mathscr U}{mathfrak g}.y.v_lambda={mathscr U}{mathfrak n}^-{mathscr U}{mathfrak b}.y.v_lambda$. In particular, there exists $xin {mathscr U}{mathfrak n}^+$ such that $(xy).v_lambdaneq 0$ in $L(lambda)_lambda$. For such $x$, we must have $(xy)_0neq 0$ since the $({mathscr U}{mathfrak g}){mathfrak n}^+$-component of $xy$ acts trivially on the highest weight vector $v_lambda$. Finally, note that $zin{mathscr U}{mathfrak h}$ acts on $v_lambda$ by $z(lambda)$.



    In the previous corollary, the roles of $x$ and $y$ can be reversed:




    Corollary: For $theta$, $lambda$ as before and $xin {mathscr U}{mathfrak n}^+_{theta}$, there exists an $yin {mathscr U}{mathfrak n}^-_{-theta}$ such that $(xy)_0(lambda)neq 0$.




    Proof: Apply the corollary to $tau(y)in {mathscr U}{mathfrak n}^-_{-theta}$, where $tau:{mathscr U}{mathfrak g}^{text{opp}}to{mathscr U}{mathfrak g}$ is the auto-involution of ${mathscr U}{mathfrak g}$ swapping ${mathfrak n}^+$ and ${mathfrak n}^-$.




    Theorem (Separation of Points): For any $zin {mathscr U}{mathfrak g}setminus{0}$ there exists a finite-dimensional $L(lambda)$ such that $z.L(lambda)neq 0$.




    Proof: Assume $z=sum_theta y_theta h_theta x_theta$ where $x_thetain{mathscr U}{mathfrak n}^+_theta$ and $y_thetain{mathscr U}{mathfrak n}^-$, $h_thetain{mathscr U}{mathfrak h}$; in other words, you group PBW terms by the weight on the ${mathfrak n}$-side. Now, consider $theta$ maximal w.r.t. the ordering $lambdapreceqmu:Leftrightarrow mu-lambdain{mathbb Z}^+Phi^+$ such that $y_theta h_theta x_theta$ nonzero. Then, we know from our previous work that there's some $lambdagg 0$ such that for any $lambda^{prime}$ such $lambdapreceqlambda^{prime}$ there exists some $y^{prime}_thetain{mathscr U}{mathfrak n}^-_{-theta}$ (depending on $lambda^{prime}$) such that $(x_theta y^{prime}_theta)_0(lambda^{prime})neq 0$. Picking $lambda^{prime}$ large enough, we may assume that also $h_theta(lambda^{prime})neq 0$; this is because the polynomial $h_thetain {mathscr P}{mathfrak h}cong{mathscr P}({mathfrak h}^{ast})$ cannot vanish on the shifted half-lattice $lambda + {mathbb Z}^+Phi^+$. Putting everything together, in $L(lambda^{prime})$ we then have $(y_theta h_theta x_theta).(y^{prime}_theta v_{lambda^{prime}}) = h_theta(lambda^{prime}) (x_theta y^{prime}_theta)_0(lambda^{prime}) y_theta v_{lambda^{prime}}neq 0$, where for the last step we potentially have to enlarge $lambda^{prime}$ again. What about the other summands in $z$? They all annihilate $y^{prime}_theta v_{lambda^{prime}}$ because of the maximality of $theta$.







    share|cite|improve this answer














    share|cite|improve this answer



    share|cite|improve this answer








    edited Nov 23 at 21:05

























    answered Nov 23 at 7:52









    Hanno

    14.7k21743




    14.7k21743












    • Your answer was superb!
      – Matheus Manzatto
      Nov 23 at 15:35


















    • Your answer was superb!
      – Matheus Manzatto
      Nov 23 at 15:35
















    Your answer was superb!
    – Matheus Manzatto
    Nov 23 at 15:35




    Your answer was superb!
    – Matheus Manzatto
    Nov 23 at 15:35










    up vote
    3
    down vote














    Theorem (Harish-Chandra 1949) for an arbitrary finite-dimensional Lie algebra over a field of characteristic, finite-dimensional representations separate points of the universal enveloping algebra.




    This is a deep result, proved in Chap 2 of Dixmier's book "enveloping algebras". It has Ado's theorem as corollary.



    In the semisimple case in characteristic zero, finite-dimensional representations split as direct sum of irreducible representations (Weyl), and hence the desired result follows: finite-dimensional irreducible representations separate points.



    I don't know if Harish-Chandra's theorem is much easier in the semisimple case (for which Ado's theorem is trivial).



    I'm not sure of the picture in finite characteristic, but this is probably tackled by work of Jacobson or so.






    share|cite|improve this answer

























      up vote
      3
      down vote














      Theorem (Harish-Chandra 1949) for an arbitrary finite-dimensional Lie algebra over a field of characteristic, finite-dimensional representations separate points of the universal enveloping algebra.




      This is a deep result, proved in Chap 2 of Dixmier's book "enveloping algebras". It has Ado's theorem as corollary.



      In the semisimple case in characteristic zero, finite-dimensional representations split as direct sum of irreducible representations (Weyl), and hence the desired result follows: finite-dimensional irreducible representations separate points.



      I don't know if Harish-Chandra's theorem is much easier in the semisimple case (for which Ado's theorem is trivial).



      I'm not sure of the picture in finite characteristic, but this is probably tackled by work of Jacobson or so.






      share|cite|improve this answer























        up vote
        3
        down vote










        up vote
        3
        down vote










        Theorem (Harish-Chandra 1949) for an arbitrary finite-dimensional Lie algebra over a field of characteristic, finite-dimensional representations separate points of the universal enveloping algebra.




        This is a deep result, proved in Chap 2 of Dixmier's book "enveloping algebras". It has Ado's theorem as corollary.



        In the semisimple case in characteristic zero, finite-dimensional representations split as direct sum of irreducible representations (Weyl), and hence the desired result follows: finite-dimensional irreducible representations separate points.



        I don't know if Harish-Chandra's theorem is much easier in the semisimple case (for which Ado's theorem is trivial).



        I'm not sure of the picture in finite characteristic, but this is probably tackled by work of Jacobson or so.






        share|cite|improve this answer













        Theorem (Harish-Chandra 1949) for an arbitrary finite-dimensional Lie algebra over a field of characteristic, finite-dimensional representations separate points of the universal enveloping algebra.




        This is a deep result, proved in Chap 2 of Dixmier's book "enveloping algebras". It has Ado's theorem as corollary.



        In the semisimple case in characteristic zero, finite-dimensional representations split as direct sum of irreducible representations (Weyl), and hence the desired result follows: finite-dimensional irreducible representations separate points.



        I don't know if Harish-Chandra's theorem is much easier in the semisimple case (for which Ado's theorem is trivial).



        I'm not sure of the picture in finite characteristic, but this is probably tackled by work of Jacobson or so.







        share|cite|improve this answer












        share|cite|improve this answer



        share|cite|improve this answer










        answered Nov 18 at 11:54









        YCor

        7,098827




        7,098827






























            draft saved

            draft discarded




















































            Thanks for contributing an answer to Mathematics Stack Exchange!


            • Please be sure to answer the question. Provide details and share your research!

            But avoid



            • Asking for help, clarification, or responding to other answers.

            • Making statements based on opinion; back them up with references or personal experience.


            Use MathJax to format equations. MathJax reference.


            To learn more, see our tips on writing great answers.





            Some of your past answers have not been well-received, and you're in danger of being blocked from answering.


            Please pay close attention to the following guidance:


            • Please be sure to answer the question. Provide details and share your research!

            But avoid



            • Asking for help, clarification, or responding to other answers.

            • Making statements based on opinion; back them up with references or personal experience.


            To learn more, see our tips on writing great answers.




            draft saved


            draft discarded














            StackExchange.ready(
            function () {
            StackExchange.openid.initPostLogin('.new-post-login', 'https%3a%2f%2fmath.stackexchange.com%2fquestions%2f3001902%2ffinite-dimensional-irreducible-representations-of-a-semisimple-lie-algebra-separ%23new-answer', 'question_page');
            }
            );

            Post as a guest















            Required, but never shown





















































            Required, but never shown














            Required, but never shown












            Required, but never shown







            Required, but never shown

































            Required, but never shown














            Required, but never shown












            Required, but never shown







            Required, but never shown







            Popular posts from this blog

            Biblatex bibliography style without URLs when DOI exists (in Overleaf with Zotero bibliography)

            ComboBox Display Member on multiple fields

            Is it possible to collect Nectar points via Trainline?